270 views
1 votes
1 votes
if L1 ={a^n | n>=0}

then L1^R  is?

1 Answer

Related questions

0 votes
0 votes
0 answers
1
RahulVerma3 asked Apr 2
52 views
Is this the correct Turing machine for the language $0^n 1^n0^n$?assuming $ at the end and begining of the input tape
0 votes
0 votes
0 answers
3
baofbuiafbi asked Nov 14, 2023
160 views
Prove the language L={(G,H)|G is a CFG, H is a DFA, and L(G)∩L(H)=∅} is undecidable.